LSAT and Law School Admissions Forum

Get expert LSAT preparation and law school admissions advice from PowerScore Test Preparation.

 ConfusedStudent
  • Posts: 2
  • Joined: Jul 02, 2018
|
#48071
Hi, I had a hard time choosing between B and C on this question. Can someone explain the difference between the two? Thanks!
User avatar
 Jonathan Evans
PowerScore Staff
  • PowerScore Staff
  • Posts: 726
  • Joined: Jun 09, 2016
|
#48145
Hi,

Great question! It is difficult to discern the difference between these two answers. The distinction is one of degree:
  • Answer choice (B) discusses what happens when scientists fail to replicate experiments. If a scientist fails to replicate a result, this failure might be due to this "riddled basin of attraction" metaphor/phenomenon.
  • Answer choice (C) makes a much stronger statement, that experimental results in general cannot be replicated because of this riddled basin of attraction phenomenon.
In other words, (B) limits itself to situations in which replication has been unsuccessful. (C) makes a broader, more extreme claim. This difference in degree provides a good tool for contrasting these two answer choices and determining which one is better supported by the text. In this case, we can find evidence in lines 54 to the end of the passage (which is where the author states the thesis): when scientists fail to replicate experiments, it might be because of this principle illustrated in the riddled basins of attraction.

This statement is a pretty good match for answer choice (B). Answer choice (C) goes beyond what the text supports and is too extreme/out of scope.

I hope this helps!
User avatar
 annabelle.swift
  • Posts: 54
  • Joined: Sep 01, 2021
|
#91947
Hi!

I think I'm struggling with understanding the analogy b/w riddled basins of attraction and replication. I understand that B is better than C since C is too extreme. However, the "physical systems" part of B threw me off. What exactly are these physical systems that make replication virtually impossible?
 Adam Tyson
PowerScore Staff
  • PowerScore Staff
  • Posts: 5153
  • Joined: Apr 14, 2011
|
#92096
To be honest, annabelle.swift, I don't know! The passage never really tells us what those physical systems are...but it doesn't have to. The fact that they are talking about physical systems (as opposed to, I suppose, theoretical ones?) is established in the first paragraph: "But physicists John Sommerer and Edward Ott have conceived of a physical system in which even the least change in the starting conditions—no matter how small, inadvertent, or undetectable—can alter results radically. " All the subsequent references to "systems" refer back to that original statement. Everything they are talking about applies to these physical systems, whatever they may be.

The best way to find the correct answer here is, as usual, to prephrase. Once you are done reading the passage you should always pause and ask yourself what the Main Point was. What does this author want us to believe? You should probably have come up with something like "these guys have an idea that, if they are right, could mean that valid scientific results might not always be capable of replication, which is a big deal." It's not about all results being that way, or even many, but just that some of them might be, and that makes answer B a great match.

Get the most out of your LSAT Prep Plus subscription.

Analyze and track your performance with our Testing and Analytics Package.